- Mon May 11, 2015 10:51 pm
#18605
Hi Art,
Thanks for your post!
This is a challenging advanced linear game (for those of you following along, we're looking at the four medical training sessions, which was on the June 1997 LSAT). The main thing that makes it difficult is that it's Unbalanced: we have four training sessions, and only three nurses and three doctors to teach them. This, combined with the first rule that "Each professional teaches at least once," means that one nurse is going to teach twice and the others will each teach once. In the language of Numerical Distributions, which we discuss in detail on page 126, this would be a 2-1-1 Unfixed distribution.
So that means that either Leopold or Fine could teach two sessions (Johnson can only teach one because of the fourth rule). That's why question 9 answer choice (C) actually isn't a direct rule violation and doesn't have to be false: it's possible that Leopold teaches both the first session and the third session.
The reason why answer choice (B) does have to be false is that the fourth rule, "Johnson teaches session S only," guarantees that S is paired with J - there's only one session S, so for any day that has session S, I can guarantee that J is going to be teaching it. Since the second rule guarantees that L is going to be teaching on day 3, it therefore can't be session S.
I hope this helps!
Thanks,
Andrew